Difference between revisions of "2002 AMC 10A Problems/Problem 20"

m (Problem)
Line 4: Line 4:
 
<math>\text{(A)}\ 5/4 \qquad \text{(B)}\ 4/3 \qquad \text{(C)}\ 3/2 \qquad \text{(D)}\ 5/3 \qquad \text{(E)}\ 2</math>
 
<math>\text{(A)}\ 5/4 \qquad \text{(B)}\ 4/3 \qquad \text{(C)}\ 3/2 \qquad \text{(D)}\ 5/3 \qquad \text{(E)}\ 2</math>
  
[asy]
+
<math>[asy]
 
pair A,B,C,D,EE,F,G,H,J;
 
pair A,B,C,D,EE,F,G,H,J;
 
A = (0,0);
 
A = (0,0);
Line 18: Line 18:
 
draw(H--C--G--D);
 
draw(H--C--G--D);
 
draw(J--EE--G);
 
draw(J--EE--G);
label("<math>A</math>",A,SW);
+
label("</math>A<math>",A,SW);
label("<math>B</math>",B,S);
+
label("</math>B<math>",B,S);
label("<math>C</math>",C,S);
+
label("</math>C<math>",C,S);
label("<math>D</math>",D,S);
+
label("</math>D<math>",D,S);
label("<math>E</math>",EE,S);
+
label("</math>E<math>",EE,S);
label("<math>F</math>",F,SE);
+
label("</math>F<math>",F,SE);
label("<math>J</math>",J,NE);
+
label("</math>J<math>",J,NE);
label("<math>G</math>",G,N);
+
label("</math>G<math>",G,N);
label(scale(0.9)*"<math>H</math>",H,NE,UnFill(0.1mm));
+
label(scale(0.9)*"</math>H<math>",H,NE,UnFill(0.1mm));
[/asy]
+
[/asy]</math>
  
 
==Solution==
 
==Solution==

Revision as of 18:24, 28 November 2015

Problem

Points $A,B,C,D,E$ and $F$ lie, in that order, on $\overline{AF}$, dividing it into five segments, each of length 1. Point $G$ is not on line $AF$. Point $H$ lies on $\overline{GD}$, and point $J$ lies on $\overline{GF}$. The line segments $\overline{HC}, \overline{JE},$ and $\overline{AG}$ are parallel. Find $HC/JE$.

$\text{(A)}\ 5/4 \qquad \text{(B)}\ 4/3 \qquad \text{(C)}\ 3/2 \qquad \text{(D)}\ 5/3 \qquad \text{(E)}\ 2$

$[asy] pair A,B,C,D,EE,F,G,H,J; A = (0,0); B = (0.2,0); C = 2*B; D = 3*B; EE = 4*B; F = 5*B; G = (-0.2,0.8); H = intersectionpoint(G--D,C -- (C + G)); J = intersectionpoint(G--F,EE--(EE+G)); draw(G--F--A--G--B); draw(H--C--G--D); draw(J--EE--G); label("$A$",A,SW); label("$B$",B,S); label("$C$",C,S); label("$D$",D,S); label("$E$",EE,S); label("$F$",F,SE); label("$J$",J,NE); label("$G$",G,N); label(scale(0.9)*"$H$",H,NE,UnFill(0.1mm)); [/asy]$

Solution

Solution #1: Since $AG$ and $CH$ are parallel, triangles $GAD$ and $HCD$ are similar. Hence, $CH/AG = CD/AD = 1/3$.

Since $AG$ and $JE$ are parallel, triangles $GAF$ and $JEF$ are similar. Hence, $EJ/AG = EF/AF = 1/5$. Therefore, $CH/EJ = (CH/AG)/(EJ/AG) = (1/3)/(1/5) = \boxed{5/3}$. The answer is (D).

Solution #2: As $\overline{JE}$ is parallel to $\overline{AG}$, angles FJE and FGA are congruent. Also, angle F is clearly congruent to itself. From AA similarity, $\triangle AGF \sim \triangle EJF$; hence $\frac {AG}{JE} =5$. Similarly, $\frac {AG}{HC} = 3$. Thus, $\frac {HC}{JE} = \boxed{\frac {5}{3}\Rightarrow \text{(D)}}$.

See Also

2002 AMC 10A (ProblemsAnswer KeyResources)
Preceded by
Problem 19
Followed by
Problem 21
1 2 3 4 5 6 7 8 9 10 11 12 13 14 15 16 17 18 19 20 21 22 23 24 25
All AMC 10 Problems and Solutions

The problems on this page are copyrighted by the Mathematical Association of America's American Mathematics Competitions. AMC logo.png